Prime Power

Three primes, represented by p p , q q , and r r , satisfy the equation below:

p q + q p = r p q { p }^{ q }+{ q }^{ p }={ r }^{ p-q }

Find the minimum value of p + q + r p+q+r .


The answer is 22.

This section requires Javascript.
You are seeing this because something didn't load right. We suggest you, (a) try refreshing the page, (b) enabling javascript if it is disabled on your browser and, finally, (c) loading the non-javascript version of this page . We're sorry about the hassle.

1 solution

Finn Hulse
Mar 30, 2014

Notice the other side of the equation r p q r^{p-q} . We can't have this equal to zero, because that would evaluate to one, which can't be expressed as the sum of prime powers. Also, it can't be negative, thus q < p q < p . Now let us examine the smallest possible cases. The two smallest primes are two and three. If we set these two values as p p and q q , then their difference is one, and r r becomes seventeen, which is prime. The answer is therefore 17 + 3 + 2 = 22 17+3+2=\boxed{22} .

How can you be sure that for other values of p , q p,q , say p = 3 , q = 5 p=3, q=5 , we can't have a small r r like r = 7 r=7 that makes the sum smaller? (Of course, ( p , q , r ) = ( 3 , 5 , 7 ) (p,q,r) = (3,5,7) doesn't satisfy, but you get the idea.)

Ivan Koswara - 7 years, 2 months ago

Log in to reply

r 2 r \geqslant 2 \Rightarrow p > q p>q

First : If q > 2 q>2

q p q^{p} is odd & p q p^{q} is odd too \rightarrow q p + p q 2 N \frac{q^{p}+p^{q}}{2}\in N \rightarrow r=2 \rightarrow p q + q p = 2 p q p^{q}+q^{p}=2^{p-q} but q p > 2 p q q^{p} >2^{p-q} \Rightarrow p q + q p > 2 p q p^{q}+q^{p} > 2^{p-q}

second : If q = 2 q=2

  • p = 3 p=3 \Rightarrow r = 17 \boxed{r=17}

  • p > 3 p>3 then p= n ± 1 n \pm 1 (n: n 3 N \frac {n}{3}\in N ) , p 2 + 2 p p^{2}+2^{p} = ( n ± 1 ) 2 + 2 p (n\pm1)^{2}+2^{p} = n 2 ± 2 n + 1 + 2 p n^{2}\pm2n+1+2^{p} , 2 p + 1 3 N \frac {2^{p}+1}{3} \in N ( p p is prime greater than 2 ) \rightarrow n 2 ± 2 n + 1 + 2 p 3 N \frac{n^{2}\pm2n+1+2^{p}}{3}\in N \rightarrow r = 3 r=3 this gives two statements:

    1 . p = 5 p=5 \Rightarrow 5 2 + 2 5 3 5 2 5^{2}+2^{5}\neq3^{5-2}

    2 . p > 5 p>5 \Rightarrow p 2 + 2 p < 3 p 2 p^{2}+2^{p}<3^{p-2}

\therefore Hence: p = 3 , q = 2 p=3,q=2 and r = 17 r=17 is the only solution

Mahdi Al-kawaz - 7 years, 2 months ago

The left side is powered to two numbers while the right is powered to their difference; q and p ramp up much faster than p-q, so r gets bigger the bigger numbers you use

Xian Ng - 7 years, 2 months ago

You're welcome to check, but I just didn't include that in my short solution. :D

Finn Hulse - 7 years, 2 months ago

Hey its just an instinct @Finn Hulse

ashutosh mahapatra - 7 years, 2 months ago

Log in to reply

Yeah. :D

Finn Hulse - 7 years, 2 months ago

0 pending reports

×

Problem Loading...

Note Loading...

Set Loading...